Đến nội dung

Hình ảnh

Chứng minh rằng tồn tại số nguyên dương $P$ squarefree sao cho $\sum_{j=1}^{n}a_jx_{i+Dk_j}=0,\forall i\in\mathbb{Z},D\equiv 1(mod\; P)$

- - - - -

Lời giải poset, 13-04-2023 - 23:04

Với đa thức nguyên $Q(X)=b_nX^n+b_{n-1}X^{n-1}+...+b_1X+b_0$, ta định nghĩa $Q(X)[0]=0,Q(X)[x_i]=b_nx_{i+n}+b_{n-1}x_{i+n-1}+...+b_1x_{i+1}+b_0x_i$ và $Q(X)[d_{i_1}x_{i_1}+d_{i_2}x_{i_2}+...+d_{i_m}x_{i_m}]=d_{i_1}Q(X)[x_{i_1}]+d_{i_2}Q(X)[x_{i_2}]+...+d_{i_m}Q(X)[x_{i_m}]$, dễ thấy định nghĩa này hoàn toàn xác định.

Bổ đề
Với hai đa thức nguyên $Q(X),R(X)$, ta có:
a) $(Q(X)+R(X))[d_{i_1}x_{i_1}+d_{i_2}x_{i_2}+...+d_{i_m}x_{i_m}]=Q(X)[d_{i_1}x_{i_1}+d_{i_2}x_{i_2}+...+d_{i_m}x_{i_m}]+R(X)[d_{i_1}x_{i_1}+d_{i_2}x_{i_2}+...+d_{i_m}x_{i_m}]$.
b) $Q(X)[R(X)[d_{i_1}x_{i_1}+d_{i_2}x_{i_2}+...+d_{i_m}x_{i_m}]]=(Q(X)R(X))[d_{i_1}x_{i_1}+d_{i_2}x_{i_2}+...+d_{i_m}x_{i_m}]$.

Chứng minh
 
a) Vì tính chất cộng tính ở định nghĩa, ta chỉ cần chứng minh $(Q(X)+R(X))[x_i]=Q(X)[x_i]+R(X)[x_i],\forall i\in\mathbb{Z}$. Giả sử $Q(X)=b_nX^n+b_{n-1}X^{n-1}+...+b_1X+b_0,R(X)=c_nX^n+c_{n-1}X^{n-1}+...+c_1X+c_0$ với $n=\max(deg(P),deg(Q))$, ta có:
$\begin{matrix}
(Q(X)+R(X))[x_i]=((b_n+c_n)X^n+(b_{n-1}+c_{n-1})X^{n-1}+...+(b_1+c_1)X+b_0+c_0)[x_i]\\
=(b_n+c_n)x_{i+n}+(b_{n-1}+c_{n-1}x_{i+n-1}+...+(b_1+c_1)x_{i+1}+(b_0+c_0)x_i\\
=(b_nx_{i+n}+b_{n-1}x_{i+n-1}+...+b_1x_{i+1}+b_0x_i)+(c_nx_{i+n}+c_{n-1}x_{i+n-1}+...+c_1x_{i+1}+c_0x_i)=Q(X)[x_i]+R(X)[x_i]
\end{matrix}$.
b) Vì tính chất cộng tính ở định nghĩa và a), ta chỉ cần chứng minh $X^n[X^m[x_i]]=X^{n+m}[x_i],\forall i\in\mathbb{Z}$, điều này hiển nhiên.

Bổ đề
Với đa thức nguyên $Q(X)$ và số nguyên tố $p$, ta có $Q^p(X)\equiv Q(X^p)(mod\; p)$.

Chứng minh
Ta có kết quả quen thuộc: $C_p^k\equiv 0(mod\; p)$ với $p$ nguyên tố và $1\leq k\leq p-1$, do đó $(X+Y)^p\equiv X^p+Y^p(mod\; p)$.
Ta quy nạp theo bậc của $Q(X)$, với $deg(Q)=0$ ta có $b^p\equiv b(mod\; p)$, đúng theo định lý Fermat nhỏ.
Giả sử bổ đề đúng với mọi đa thức có bậc nhỏ hơn $n\geq 1$, với $deg(Q)=n$ ta phân tích $Q(X)=bX^n+R(X)$ với $deg(R)<deg(Q)$. Ta có:
$Q^p(X)=(bX^n+R(X))^p\equiv b^pX^{np}+R^p(X)\equiv b(X^p)^n+R(X^p)=Q(X^p)(mod\; p)$
theo giả thuyết quy nạp với đa thức $R(X)$ và định lý Fermat nhỏ. 
Vậy bổ đề đã được chứng minh.

Quay trở lại bài toán. Bằng cách cộng thêm vào các số $k_i$ cùng một số đủ lớn, bài toán vẫn không thay đổi và ta có thể giả sử $k_i$ không âm. Ta đặt $Q(X)=a_1x^{k_1}+a_2x^{k_2}+...+a_nx^{k_n}$ là một đa thức nguyên, theo giả thuyết đề bài ta có $Q(X)[x_i]=0,\forall i\in\mathbb{Z}$. Ta đặt $N=\max_{1\leq i\leq n}(a_i)$. Ta sẽ chứng minh với mọi số nguyên dương $D$ có mọi ước nguyên tố lớn hơn $MNn$, ta có $\sum_{j=1}^na_jx_{i+Dk_j}=Q(X^D)[x_i]=0,\forall i\in\mathbb{Z}$. Ta sẽ chứng minh bài toán mạnh hơn, với mọi đa thức nguyên $R(X)$ có hệ số bị chặn bởi $N$ và số hệ số khác $0$ không quá $n$ thỏa mãn $R(X)[x_i]=0,\forall i\in\mathbb{Z}$, ta có $R(X^D)[x_i]=0,\forall i\in\mathbb{Z}$.
Với $D=p_1^{q_1}p_2^{q_2}...p_m^{q_m},p_i>MNn$, định nghĩa $deg(D)=\sum_{i=1}^mq_i$. Ta sẽ chứng minh bài toán bằng quy nạp theo $deg(D)$:

-Với $deg(D)=0$, tức $D=1$ thì $R(X^D)[x_i]=R(X)[x_i]=0,\forall i\in\mathbb{Z}$ đúng.

-Với $deg(D)=1$, tức $D=p$ với $p>MNn$ là số nguyên tố. Theo Theorem, tồn tại đa thức nguyên $S(X)$ thỏa mãn $R^p(X)=R(X^p)+pS(X)$. Theo Theorem và giả thuyết bài toán ta có:
$0=R^{p-1}(X)[0]=R^{p-1}[R(X)[x_i]]=R^p(X)[x_i]=(R(X^p)+pS(X))[x_i]=R(X^p)[x_i]+pS(X)[x_i],\forall i\in\mathbb{Z}$
tức $R(X^p)[x_i]\equiv 0(mod\; p),\forall i\in\mathbb{Z}$. Ta lại có:
$\left | R(X^p)[x_i] \right |=\left | \sum_{j=1}^{n}b_jx_{i+pl_j} \right |\leq\sum_{j=1}^{n}|b_jx_{i+pl_j}|\leq MNn<p,\forall i\in\mathbb{Z}$ theo giả thuyết của đề bài và đa thức $R(X)$, do đó $R(X^p)[x_i]=0,\forall i\in\mathbb{Z}$, đúng.

-Với $deg(D)>1$, giả sử bài toán đúng với $E$ có mọi ước nguyên tố lớn hơn $MNn$ và $deg(E)=deg(D)-1$. Chọn $p|D,p>MNn$ và $D=pE$ ta có $deg(E)=deg(D)-1$ và $E$ có mọi ước nguyên tố lớn hơn $MNn$, theo giả thuyết quy nạp với $E$ ta có $R(X^E)[x_i]=0,\forall i\in\mathbb{Z}$, ta có $R(X^E)$ có hệ số bị chặn bởi $N$ và số hệ số khác $0$ không quá $n$, do vậy áp dụng bài toán trong trường hợp $p>MNn$ đã chứng minh ở trên ta có $R(X^D)[x_i]=R((X^p)^E)[x_i]=0,\forall i\in\mathbb{Z}$.
Vậy bài toán được chứng minh.

Với $D$ âm và có mọi ước nguyên tố lớn hơn $MNn$, ta đổi dấu $D,k_1,k_2,...,k_n$ và làm tương tự ta cũng có $\sum_{j=1}^na_jx_{i+Dk_j}=Q(X^D)[x_i]=0,\forall i\in\mathbb{Z}$.
Ta chọn $P$ là tích của tất cả số nguyên tố không vượt quá $\max(MNn,2)$ thì $P$ squarefree và nếu $D\equiv 1(mod\; P)$ thì $D$ có mọi ước nguyên tố lớn hơn $MNn$, do đó $\sum_{j=1}^na_jx_{i+Dk_j}=0,\forall i\in\mathbb{Z}$. Vậy $P$ thỏa mãn đề bài.
 

Đi đến bài viết »


  • Please log in to reply
Chủ đề này có 1 trả lời

#1
poset

poset

    Trung sĩ

  • ĐHV Toán Cao cấp
  • 125 Bài viết
Cho dãy số nguyên $...,x_{-2},x_{-1},x_0,x_1,x_2,...$ bị chặn (tức tồn tại $M$ sao cho $|x_i|<M,\forall i\in \mathbb{Z}$). Giả sử tồn tại các số nguyên $a_1,...,a_n,k_1,...,k_n$ sao cho $\sum_{j=1}^{n}a_jx_{i+k_j}=0,\forall i\in\mathbb{Z}$. Chứng minh rằng tồn tại số nguyên dương $P$ squarefree sao cho $\sum_{j=1}^{n}a_jx_{i+Dk_j}=0,\forall i\in\mathbb{Z},D\equiv 1(mod\; P)$.

Bài viết đã được chỉnh sửa nội dung bởi poset: 10-10-2022 - 12:03


#2
poset

poset

    Trung sĩ

  • ĐHV Toán Cao cấp
  • 125 Bài viết
✓  Lời giải

Với đa thức nguyên $Q(X)=b_nX^n+b_{n-1}X^{n-1}+...+b_1X+b_0$, ta định nghĩa $Q(X)[0]=0,Q(X)[x_i]=b_nx_{i+n}+b_{n-1}x_{i+n-1}+...+b_1x_{i+1}+b_0x_i$ và $Q(X)[d_{i_1}x_{i_1}+d_{i_2}x_{i_2}+...+d_{i_m}x_{i_m}]=d_{i_1}Q(X)[x_{i_1}]+d_{i_2}Q(X)[x_{i_2}]+...+d_{i_m}Q(X)[x_{i_m}]$, dễ thấy định nghĩa này hoàn toàn xác định.

Bổ đề
Với hai đa thức nguyên $Q(X),R(X)$, ta có:
a) $(Q(X)+R(X))[d_{i_1}x_{i_1}+d_{i_2}x_{i_2}+...+d_{i_m}x_{i_m}]=Q(X)[d_{i_1}x_{i_1}+d_{i_2}x_{i_2}+...+d_{i_m}x_{i_m}]+R(X)[d_{i_1}x_{i_1}+d_{i_2}x_{i_2}+...+d_{i_m}x_{i_m}]$.
b) $Q(X)[R(X)[d_{i_1}x_{i_1}+d_{i_2}x_{i_2}+...+d_{i_m}x_{i_m}]]=(Q(X)R(X))[d_{i_1}x_{i_1}+d_{i_2}x_{i_2}+...+d_{i_m}x_{i_m}]$.

Chứng minh
 
a) Vì tính chất cộng tính ở định nghĩa, ta chỉ cần chứng minh $(Q(X)+R(X))[x_i]=Q(X)[x_i]+R(X)[x_i],\forall i\in\mathbb{Z}$. Giả sử $Q(X)=b_nX^n+b_{n-1}X^{n-1}+...+b_1X+b_0,R(X)=c_nX^n+c_{n-1}X^{n-1}+...+c_1X+c_0$ với $n=\max(deg(P),deg(Q))$, ta có:
$\begin{matrix}
(Q(X)+R(X))[x_i]=((b_n+c_n)X^n+(b_{n-1}+c_{n-1})X^{n-1}+...+(b_1+c_1)X+b_0+c_0)[x_i]\\
=(b_n+c_n)x_{i+n}+(b_{n-1}+c_{n-1}x_{i+n-1}+...+(b_1+c_1)x_{i+1}+(b_0+c_0)x_i\\
=(b_nx_{i+n}+b_{n-1}x_{i+n-1}+...+b_1x_{i+1}+b_0x_i)+(c_nx_{i+n}+c_{n-1}x_{i+n-1}+...+c_1x_{i+1}+c_0x_i)=Q(X)[x_i]+R(X)[x_i]
\end{matrix}$.
b) Vì tính chất cộng tính ở định nghĩa và a), ta chỉ cần chứng minh $X^n[X^m[x_i]]=X^{n+m}[x_i],\forall i\in\mathbb{Z}$, điều này hiển nhiên.

Bổ đề
Với đa thức nguyên $Q(X)$ và số nguyên tố $p$, ta có $Q^p(X)\equiv Q(X^p)(mod\; p)$.

Chứng minh
Ta có kết quả quen thuộc: $C_p^k\equiv 0(mod\; p)$ với $p$ nguyên tố và $1\leq k\leq p-1$, do đó $(X+Y)^p\equiv X^p+Y^p(mod\; p)$.
Ta quy nạp theo bậc của $Q(X)$, với $deg(Q)=0$ ta có $b^p\equiv b(mod\; p)$, đúng theo định lý Fermat nhỏ.
Giả sử bổ đề đúng với mọi đa thức có bậc nhỏ hơn $n\geq 1$, với $deg(Q)=n$ ta phân tích $Q(X)=bX^n+R(X)$ với $deg(R)<deg(Q)$. Ta có:
$Q^p(X)=(bX^n+R(X))^p\equiv b^pX^{np}+R^p(X)\equiv b(X^p)^n+R(X^p)=Q(X^p)(mod\; p)$
theo giả thuyết quy nạp với đa thức $R(X)$ và định lý Fermat nhỏ. 
Vậy bổ đề đã được chứng minh.

Quay trở lại bài toán. Bằng cách cộng thêm vào các số $k_i$ cùng một số đủ lớn, bài toán vẫn không thay đổi và ta có thể giả sử $k_i$ không âm. Ta đặt $Q(X)=a_1x^{k_1}+a_2x^{k_2}+...+a_nx^{k_n}$ là một đa thức nguyên, theo giả thuyết đề bài ta có $Q(X)[x_i]=0,\forall i\in\mathbb{Z}$. Ta đặt $N=\max_{1\leq i\leq n}(a_i)$. Ta sẽ chứng minh với mọi số nguyên dương $D$ có mọi ước nguyên tố lớn hơn $MNn$, ta có $\sum_{j=1}^na_jx_{i+Dk_j}=Q(X^D)[x_i]=0,\forall i\in\mathbb{Z}$. Ta sẽ chứng minh bài toán mạnh hơn, với mọi đa thức nguyên $R(X)$ có hệ số bị chặn bởi $N$ và số hệ số khác $0$ không quá $n$ thỏa mãn $R(X)[x_i]=0,\forall i\in\mathbb{Z}$, ta có $R(X^D)[x_i]=0,\forall i\in\mathbb{Z}$.
Với $D=p_1^{q_1}p_2^{q_2}...p_m^{q_m},p_i>MNn$, định nghĩa $deg(D)=\sum_{i=1}^mq_i$. Ta sẽ chứng minh bài toán bằng quy nạp theo $deg(D)$:

-Với $deg(D)=0$, tức $D=1$ thì $R(X^D)[x_i]=R(X)[x_i]=0,\forall i\in\mathbb{Z}$ đúng.

-Với $deg(D)=1$, tức $D=p$ với $p>MNn$ là số nguyên tố. Theo Theorem, tồn tại đa thức nguyên $S(X)$ thỏa mãn $R^p(X)=R(X^p)+pS(X)$. Theo Theorem và giả thuyết bài toán ta có:
$0=R^{p-1}(X)[0]=R^{p-1}[R(X)[x_i]]=R^p(X)[x_i]=(R(X^p)+pS(X))[x_i]=R(X^p)[x_i]+pS(X)[x_i],\forall i\in\mathbb{Z}$
tức $R(X^p)[x_i]\equiv 0(mod\; p),\forall i\in\mathbb{Z}$. Ta lại có:
$\left | R(X^p)[x_i] \right |=\left | \sum_{j=1}^{n}b_jx_{i+pl_j} \right |\leq\sum_{j=1}^{n}|b_jx_{i+pl_j}|\leq MNn<p,\forall i\in\mathbb{Z}$ theo giả thuyết của đề bài và đa thức $R(X)$, do đó $R(X^p)[x_i]=0,\forall i\in\mathbb{Z}$, đúng.

-Với $deg(D)>1$, giả sử bài toán đúng với $E$ có mọi ước nguyên tố lớn hơn $MNn$ và $deg(E)=deg(D)-1$. Chọn $p|D,p>MNn$ và $D=pE$ ta có $deg(E)=deg(D)-1$ và $E$ có mọi ước nguyên tố lớn hơn $MNn$, theo giả thuyết quy nạp với $E$ ta có $R(X^E)[x_i]=0,\forall i\in\mathbb{Z}$, ta có $R(X^E)$ có hệ số bị chặn bởi $N$ và số hệ số khác $0$ không quá $n$, do vậy áp dụng bài toán trong trường hợp $p>MNn$ đã chứng minh ở trên ta có $R(X^D)[x_i]=R((X^p)^E)[x_i]=0,\forall i\in\mathbb{Z}$.
Vậy bài toán được chứng minh.

Với $D$ âm và có mọi ước nguyên tố lớn hơn $MNn$, ta đổi dấu $D,k_1,k_2,...,k_n$ và làm tương tự ta cũng có $\sum_{j=1}^na_jx_{i+Dk_j}=Q(X^D)[x_i]=0,\forall i\in\mathbb{Z}$.
Ta chọn $P$ là tích của tất cả số nguyên tố không vượt quá $\max(MNn,2)$ thì $P$ squarefree và nếu $D\equiv 1(mod\; P)$ thì $D$ có mọi ước nguyên tố lớn hơn $MNn$, do đó $\sum_{j=1}^na_jx_{i+Dk_j}=0,\forall i\in\mathbb{Z}$. Vậy $P$ thỏa mãn đề bài.
 


Bài viết đã được chỉnh sửa nội dung bởi poset: 13-04-2023 - 23:17





1 người đang xem chủ đề

0 thành viên, 1 khách, 0 thành viên ẩn danh